- Sat Oct 21, 2017 9:22 am
#40798
Complete Question Explanation
(The complete setup for this game can be found here: lsat/viewtopic.php?t=4676)
The correct answer choice is (D)
This question is made considerably easier by having the two templates.
Answer choice (A) is incorrect because, as shown in Template #1, L could dive last.
Answer choice (B) is incorrect because, as shown in Template #2, O could dive last.
Answer choice (C) is incorrect because P could dive fifth in Template #1 or #2.
Answer choice (D) is the correct answer. W must always dive after T, and because L or O always dives last, at least two members always dive after T.
Answer choice (E) is incorrect because W could dive fifth in Template #1 or #2.
(The complete setup for this game can be found here: lsat/viewtopic.php?t=4676)
The correct answer choice is (D)
This question is made considerably easier by having the two templates.
Answer choice (A) is incorrect because, as shown in Template #1, L could dive last.
Answer choice (B) is incorrect because, as shown in Template #2, O could dive last.
Answer choice (C) is incorrect because P could dive fifth in Template #1 or #2.
Answer choice (D) is the correct answer. W must always dive after T, and because L or O always dives last, at least two members always dive after T.
Answer choice (E) is incorrect because W could dive fifth in Template #1 or #2.